Những câu hỏi liên quan
dam thu a
Xem chi tiết
Nguyễn Việt Lâm
15 tháng 2 2020 lúc 13:28

\(abc+ab+bc+ca=2\)

\(\Leftrightarrow abc+ab+bc+ca+a+b+c+1=a+b+c+3\)

\(\Leftrightarrow\left(a+1\right)\left(b+1\right)\left(c+1\right)=a+b+c+3\)

\(\Leftrightarrow\frac{1}{\left(a+1\right)\left(b+1\right)}+\frac{1}{\left(b+1\right)\left(c+1\right)}+\frac{1}{\left(c+1\right)\left(a+1\right)}=1\)

Đặt \(\left(\frac{1}{a+1};\frac{1}{b+1};\frac{1}{c+1}\right)=\left(x;y;z\right)\Rightarrow xy+yz+zx=1\)

\(P=\sum\frac{x}{x^2+1}=\sum\frac{x}{\left(x+y\right)\left(x+z\right)}=\frac{2\left(xy+yz+zx\right)}{\left(x+y\right)\left(y+z\right)\left(z+x\right)}=\frac{2}{\left(x+y\right)\left(y+z\right)\left(z+x\right)}\)

Mặt khác \(\left(x+y\right)\left(y+z\right)\left(z+x\right)\ge\frac{8}{9}\left(x+y+z\right)\left(xy+yz+zx\right)=\frac{8}{9}\left(x+y+z\right)\)

\(\Rightarrow P\le\frac{9}{4\left(x+y+z\right)}\le\frac{9}{4\sqrt{3\left(xy+yz+zx\right)}}=\frac{3\sqrt{3}}{4}\)

Bình luận (0)
 Khách vãng lai đã xóa
Xem chi tiết
The Angry
4 tháng 10 2020 lúc 21:21

ĐK : \(x\in N\left|x\inℕ^∗\right|min=1\)

\(\frac{a^2b}{ab^2+1}+\frac{b^2c}{bc^2+1}+\frac{c^2a}{ca^2+1}\ge\frac{3abc}{1+abc}\)

\(\frac{1^2.1}{1.1^2+1}+\frac{1^2.1}{1.1^2+1}+\frac{1^2.1}{1.1^2+1}\ge\frac{3.1.1.1}{1+1.1.1}\)

\(\frac{2}{2}+\frac{2}{2}+\frac{2}{2}\ge\frac{3}{2}\)

\(3\ne\frac{3}{2}\)(đpcm)

Bình luận (0)
 Khách vãng lai đã xóa
Hoàng Quốc Tuấn
Xem chi tiết
tth_new
7 tháng 1 2020 lúc 18:36

4/ Xét hiệu: \(P-2\left(ab+7bc+ca\right)\)

\(=5a^2+11b^2+5c^2-2\left(ab+7bc+ca\right)\)

\(=\frac{\left(5a-b-c\right)^2+6\left(3b-2c\right)^2}{5}\ge0\)

Vì vậy: \(P\ge2\left(ab+7bc+ca\right)=2.188=376\)

Đẳng thức xảy ra khi ...(anh giải nốt ạ)

Bình luận (0)
 Khách vãng lai đã xóa
tth_new
7 tháng 1 2020 lúc 20:28

@Cool Kid:

Bài 5: Bản chất của bài này là tìm k (nhỏ nhất hay lớn nhất gì đó, mình nhớ không rõ nhưng đại khái là chọn k) sao cho: \(5a^2+11b^2+5c^2\ge k\left(ab+7bc+ca\right)\)

Rồi đó, chuyển vế, viết lại dưới dạng tam thức bậc 2 biến a, b, c gì cũng được rồi tự làm đi:)

Bình luận (0)
 Khách vãng lai đã xóa
tth_new
7 tháng 1 2020 lúc 20:29

í lộn, bài 4:v Bài 3 thấy quen quen, đợi chút em lục lại@Hoàng Quốc Tuấn 

Bình luận (0)
 Khách vãng lai đã xóa
oooloo
Xem chi tiết
Nguyễn Việt Lâm
20 tháng 6 2020 lúc 22:29

\(\sqrt{\frac{ab+2c^2}{1+ab-c^2}}=\sqrt{\frac{ab+2c^2}{a^2+b^2+ab}}=\frac{ab+2c^2}{\sqrt{\left(ab+2c^2\right)\left(a^2+b^2+ab\right)}}\ge\frac{2\left(ab+2c^2\right)}{a^2+b^2+2ab+2c^2}\ge\frac{ab+2c^2}{a^2+b^2+c^2}=ab+2c^2\)

Tương tự: \(\sqrt{\frac{bc+2a^2}{1+bc-a^2}}\ge bc+2a^2\) ; \(\sqrt{\frac{ca+2b^2}{1+ac-b^2}}\ge ca+2b^2\)

Cộng vế với vế:

\(VT\ge2\left(a^2+b^2+c^2\right)+ab+bc+ca=2+ab+bc+ca\)

Bình luận (0)
Fire Sky
Xem chi tiết
tth_new
14 tháng 11 2019 lúc 13:46

\(\Leftrightarrow\Sigma_{cyc}\frac{\left(ab+bc+ca\right)^2}{2a^2+bc}\le\left(a+b+c\right)^2\)

Ta có: \(\frac{\left(ab+bc+ca\right)^2}{2a^2+bc}\le\frac{\left(ab+ca\right)^2}{2a^2}+\frac{\left(bc\right)^2}{bc}=\frac{\left(b+c\right)^2}{2}+bc\)

Tương tự rồi cộng lại ta thu được:

\(L.H.S\le\frac{\left(a+b\right)^2+\left(b+c\right)^2+\left(c+a\right)^2}{2}+ab+bc+ca\)

\(=\frac{2\left(a^2+b^2+c^2\right)+2\left(ab+bc+ca\right)}{2}+ab+bc+ca\)\(=\left(a+b+c\right)^2\)

P/s: Nhìn đơn giản chứ nó là bao nhiêu ngày suy nghĩ đấy ạ:( Chả biết đúng hay sai nữa:v

Bình luận (0)
 Khách vãng lai đã xóa
An Vy
Xem chi tiết
bach nhac lam
Xem chi tiết
Ngô Bá Hùng
18 tháng 11 2019 lúc 20:54

1. Vai trò a, b, c như nhau. Không mất tính tổng quát. Giả sử \(a\ge b\ge0\)

\(ab+bc+ca=3\). Do đó \(ab\ge1\)

Ta cần chứng minh rằng \(\frac{1}{1+a^2}+\frac{1}{1+b^2}\ge\frac{2}{1+ab}\left(1\right)\)

\(\frac{2}{1+ab}+\frac{1}{1+c^2}\ge\frac{3}{2}\left(2\right)\)

Thật vậy: \(\left(1\right)\Leftrightarrow\frac{1}{1+a^2}-\frac{1}{1+ab}+\frac{1}{1+b^2}-\frac{1}{1+ab}\ge0\\ \Leftrightarrow\left(ab-a^2\right)\left(1+b^2\right)+\left(ab-b^2\right)\left(1+a^2\right)\ge0\\ \Leftrightarrow\left(a-b\right)\left[-a\left(1+b^2\right)+b\left(1+a^2\right)\right]\ge0\\ \Leftrightarrow\left(a-b\right)^2\left(ab-1\right)\ge0\left(BĐT:đúng\right)\)

\(\left(2\right)\Leftrightarrow c^2+3-ab\ge3abc^2\\ \Leftrightarrow c^2+ca+bc\ge3abc^2\Leftrightarrow a+b+c\ge3abc\)

BĐT đúng, vì \(\left(a+b+c\right)^2>3\left(ab+bc+ca\right)=q\)

\(ab+bc+ca\ge3\sqrt[3]{\left(abc\right)^2}\)

Nên \(a+b+c\ge3\ge3abc\)

Từ (1) và (2) ta có \(\frac{1}{1+a^2}+\frac{1}{1+b^2}+\frac{1}{1+c^2}\ge\frac{3}{2}\)

Dấu ''='' xảy ra \(\Leftrightarrow a=b=c=1\)

Bình luận (0)
 Khách vãng lai đã xóa
Ngô Bá Hùng
18 tháng 11 2019 lúc 21:16

Áp dụng BĐT Cauchy dạng \(\frac{9}{x+y+z}\le\frac{1}{x}+\frac{1}{y}+\frac{1}{z}\), ta được

\(\frac{9}{a+3b+2c}=\frac{1}{a+c+b+c+2b}\le\frac{1}{9}\left(\frac{1}{a+c}+\frac{1}{b+c}+\frac{1}{2b}\right)\)

Do đó ta được

\(\frac{ab}{a+3b+2c}\le\frac{ab}{9}\left(\frac{1}{a+c}+\frac{1}{b+c}+\frac{1}{2b}\right)=\frac{1}{9}\left(\frac{ab}{a+c}+\frac{ab}{b+c}+\frac{a}{2}\right)\)

Hoàn toàn tương tự ta được

\(\frac{bc}{2a+b+3c}\le\frac{1}{9}\left(\frac{bc}{a+b}+\frac{bc}{b+c}+\frac{b}{2}\right);\frac{ac}{3a+2b+c}\le\frac{1}{9}\left(\frac{ac}{a+b}+\frac{ac}{b+c}+\frac{c}{2}\right)\)

Cộng theo vế các BĐT trên ta được

\(\frac{ab}{a+3b+2c}+\frac{bc}{b+3c+2a}+\frac{ca}{c+3a+2b}\le\frac{1}{9}\left(\frac{ac+bc}{a+b}+\frac{ab+ac}{b+c}+\frac{bc+ab}{a+c}+\frac{a+b+c}{2}\right)=\frac{a+b+c}{6}\)Vậy BĐT đc CM

ĐẲng thức xảy ra khi và chỉ khi a = b = c >0

Bình luận (0)
 Khách vãng lai đã xóa
Akai Haruma
18 tháng 11 2019 lúc 22:38

Bài 2:

Áp dụng BĐT AM-GM:

\(a^2+2b^2+c^2=(a^2+b^2)+(a^2+c^2)\geq 2\sqrt{(a^2+b^2)(a^2+c^2)}\geq 2\sqrt{\frac{(a+b)^2}{2}.\frac{(a+c)^2}{2}}=(a+b)(a+c)\)

\(\Rightarrow \frac{ab^2}{a^2+2b^2+c^2}\leq \frac{ab^2}{(a+b)(a+c)}\)

Hoàn toàn tương tự với các phân thức còn lại:

\(\Rightarrow \text{VT}\leq \sum \frac{ab^2}{(a+b)(a+c)}=\frac{a^2b^2+b^2c^2+c^2a^2+abc(a+b+c)}{(a+b)(b+c)(c+a)}\)

Ta cần CM: \(\frac{a^2b^2+b^2c^2+c^2a^2+abc(a+b+c)}{(a+b)(b+c)(c+a)}\leq \frac{a+b+c}{4}\)

\(\Leftrightarrow 4(a^2b^2+b^2c^2+c^2a^2)+4abc(a+b+c)\leq (a+b+c)(a+b)(b+c)(c+a)\)

\(\Leftrightarrow 4(a^2b^2+b^2c^2+c^2a^2)+4abc(a+b+c)\leq (a+b+c)(a+b)(b+c)(c+a)\)

\(\Leftrightarrow 4(a^2b^2+b^2c^2+c^2a^2)+4abc(a+b+c)\leq (a+b+c)[(a+b+c)(ab+bc+ac)-abc]\)

\(\Leftrightarrow 2(a^2b^2+b^2c^2+c^2a^2)\leq (a^3b+ab^3)+(bc^3+b^3c)+(ca^3+c^3a)\)

(dễ thấy luôn đúng do theo BĐT AM-GM)

Do đó ta có đpcm.

Dấu "=" xảy ra khi $a=b=c$

Bình luận (0)
 Khách vãng lai đã xóa
nam do
Xem chi tiết
tthnew
20 tháng 7 2019 lúc 18:22

Em chỉ giải ra được 1 TH dấu bằng thôi: a = b = c (còn trường hợp a = b; c=0 và các hoán vị thì em chịu, vì khi xét dấu = trong bđt thì em chỉ xảy ra 1 th)

Áp dụng BĐT Cauchy-Schwarz dạng Engel;

\(VT\ge\frac{16}{a^2+b^2+c^2+\left(a+b+c\right)^2}\ge\frac{16}{\frac{\left(a+b+c\right)^2}{3}+\left(a+b+c\right)^2}\)\(=\frac{12}{\left(a+b+c\right)^2}\) (đpcm)

Đẳng thức xảy ra khi a = b = c

Bình luận (4)
Neet
Xem chi tiết
Akai Haruma
2 tháng 3 2017 lúc 0:34

Bài 3)

BĐT cần chứng minh tương đương với:

\(\left ( \frac{a}{a+b} \right )^2+\left ( \frac{b}{b+c} \right )^2+\left ( \frac{c}{c+a} \right )^2\geq \frac{1}{2}\left ( 3-\frac{a}{a+b}-\frac{b}{b+c}-\frac{c}{c+a} \right )\)

Để cho gọn, đặt \((x,y,z)=\left (\frac{b}{a},\frac{c}{b},\frac{a}{c}\right)\) \(\Rightarrow xyz=1\).

BĐT được viết lại như sau:

\(A=2\left [ \frac{1}{(x+1)^2}+\frac{1}{(y+1)^2}+\frac{1}{(z+1)^2} \right ]+\frac{1}{x+1}+\frac{1}{y+1}+\frac{1}{z+1}\geq 3\) \((\star)\)

Ta nhớ đến hai bổ đề khá quen thuộc sau:

Bổ đề 1: Với \(a,b>0\) thì \(\frac{1}{(a+1)^2}+\frac{1}{(b+1)^2}\geq \frac{1}{ab+1}\)

Cách CM rất đơn giản, Cauchy - Schwarz:

\((a+1)^2\leq (a+b)(a+\frac{1}{b})\Rightarrow \frac{1}{(a+1)^2}\geq \frac{b}{(a+b)(ab+1)}\)

Tương tự với biểu thức còn lại và cộng vào thu được đpcm

Bổ đề 2: Với \(x,y>0,xy\geq 1\) thì \(\frac{1}{x^2+1}+\frac{1}{y^2+1}\geq \frac{2}{xy+1}\)

Cách CM: Quy đồng ta có đpcm.

Do tính hoán vị nên không mất tổng quát giả sử \(z=\min (x,y,z)\)

\(\Rightarrow xy\geq 1\). Áp dụng hai bổ đề trên:

\(A\geq 2\left [ \frac{1}{xy+1}+\frac{1}{(z+1)^2} \right ]+\frac{2}{\sqrt{xy}+1}+\frac{1}{z+1}=2\left [ \frac{z}{z+1}+\frac{1}{(z+1)^2} \right ]+\frac{2\sqrt{z}}{\sqrt{z}+1}+\frac{1}{z+1}\)

\(\Leftrightarrow A\geq \frac{2(z^2+z+1)}{(z+1)^2}+\frac{1}{z+1}+2-\frac{2}{\sqrt{z}+1}\geq 3\)

\(\Leftrightarrow 2\left [ \frac{z^2+z+1}{(z+1)^2}-\frac{3}{4} \right ]+\frac{1}{z+1}-\frac{1}{2}-\left ( \frac{2}{\sqrt{z}+1}-1 \right )\geq 0\)

\(\Leftrightarrow \frac{(z-1)^2}{2(z+1)^2}-\frac{z-1}{2(z+1)}+\frac{z-1}{(\sqrt{z}+1)^2}\geq 0\Leftrightarrow (z-1)\left [ \frac{1}{(\sqrt{z}+1)^2}-\frac{1}{(z+1)^2} \right ]\geq 0\)

\(\Leftrightarrow \frac{\sqrt{z}(\sqrt{z}-1)^2(\sqrt{z}+1)(z+\sqrt{z}+2)}{(\sqrt{z}+1)^2(z+1)^2}\geq 0\) ( luôn đúng với mọi \(z>0\) )

Do đó \((\star)\) được cm. Bài toán hoàn tất.

Dấu bằng xảy ra khi \(a=b=c\)

P/s: Nghỉ tuyển lâu rồi giờ mới gặp mấy bài BĐT phải động não. Khuya rồi nên xin phép làm bài 3 trước. Hai bài kia xin khiếu. Nếu làm đc chắc tối mai sẽ post.

Bình luận (1)
Lightning Farron
2 tháng 3 2017 lúc 18:11

Bài 1:

Cho \(a=b=c=\dfrac{1}{\sqrt{3}}\). Khi đó \(M=\sqrt{3}-2\)

Ta sẽ chứng minh nó là giá trị nhỏ nhất

Thật vậy, đặt c là giá trị nhỏ nhất của a,b,c. Khi đó, ta cần chứng minh

\(\frac{a^2}{b}+\frac{b^2}{c}+\frac{c^2}{a}-\frac{2(a^2+b^2+c^2)}{\sqrt{ab+ac+bc}}\geq(\sqrt3-2)\sqrt{ab+ac+bc}\)

\(\Leftrightarrow\sqrt{ab+ac+bc}\left(\frac{a^2}{b}+\frac{b^2}{c}+\frac{c^2}{a}-\sqrt{3(ab+ac+bc)}\right)\geq2(a^2+b^2+c^2-ab-ac-bc)\)

\(\Leftrightarrow\frac{a^2}{b}+\frac{b^2}{a}-a-b+\frac{b^2}{c}+\frac{c^2}{a}-\frac{b^2}{a}-c+a+b+c-\sqrt{3(ab+ac+bc)}\geq\)

\(\geq2((a-b)^2+(c-a)(c-b))\)

\(\Leftrightarrow(a-b)^2\left(\frac{1}{a}+\frac{1}{b}-2\right)+(c-a)(c-b)\left(\frac{1}{a}+\frac{b}{ac}-2\right)+a+b+c-\sqrt{3(ab+ac+bc)}\geq0\)

Đúng bởi \(\frac{1}{a}+\frac{1}{b}-2>0;\frac{1}{a}+\frac{b}{ac}-2\geq\frac{1}{a}+\frac{1}{a}-2>0\)

\(a+b+c-\sqrt{3(ab+ac+bc)}=\frac{(a-b)^2+(c-a)(c-b)}{a+b+c+\sqrt{3(ab+ac+bc)}}\geq0\)

BĐT đã được c/m. Vậy \(M_{Min}=\sqrt{3}-2\Leftrightarrow a=b=c=\dfrac{1}{\sqrt{3}}\)

P/s: Nhìn qua thấy ngon mà làm mới thấy thật sự là "choáng"

Bình luận (3)
Hung nguyen
2 tháng 3 2017 lúc 10:57

Câu 1/ Ta có

\(ab+bc+ca\le\frac{\left(a+b+c\right)^2}{3}\)

\(\Leftrightarrow1\le\frac{\left(a+b+c\right)^2}{3}\)

\(\Leftrightarrow\left(a+b+c\right)^2\ge3\)

\(\Leftrightarrow\sqrt{3}\le a+b+c< 3\)

Ta có: \(M=\frac{a^2\left(1-2b\right)}{b}+\frac{b^2\left(1-2c\right)}{c}+\frac{c^2\left(1-2a\right)}{a}\)

\(=\frac{a^2}{b}+\frac{b^2}{c}+\frac{c^2}{a}-2\left(a^2+b^2+c^2\right)\)

\(\ge\frac{\left(a+b+c\right)^2}{a+b+c}-2\left(a^2+b^2+c^2\right)-4\left(ab+bc+ca\right)+4\left(ab+bc+ca\right)\)

\(=a+b+c-2\left(a+b+c\right)^2+4\) (1)

Đặt \(a+b+c=x\left(\sqrt{3}\le x< 3\right)\)

Ta tìm GTNN của hàm số: \(y=-2x^2+x+4\)

\(\Rightarrow y'=-4x+1=0\)

\(\Rightarrow x=\frac{1}{4}=0,25\)

Thế x lần lược các giá trị \(\left\{\begin{matrix}x=0,25\\x=\sqrt{3}\end{matrix}\right.\)

\(\Rightarrow\left\{\begin{matrix}y=4,125\\y=-2+\sqrt{3}\end{matrix}\right.\)

\(\Rightarrow y_{min}=-2+\sqrt{3}\) đạt cực trị tại \(x=\sqrt{3}\) (2)

Từ (1) và (2) ta suy ra GTNN của M là \(-2+\sqrt{3}\) tại \(a=b=c=\frac{1}{\sqrt{3}}\)

Bình luận (7)